The product of a number and the number decreased by 4 is 60. write a quadratic equation that models the verbal sentence. let n represent the number find the number

Answers

Answer 1

Answer:

n = 2 + (60/2)

Step-by-step explanation:


Related Questions

Joan is 5 years older than Ellen, and 3 years ago their sum of their ages is 17, in how many years will Joan be 21?

Answers

Answer:

7

Step-by-step explanation:

What should be subtracted from 3x^4 - 2x^3 + 2x^2 + 3x - 2x + 3, so that the resulting polynomial is exactly​

Answers

Answer:

185/27

Step-by-step explanation:

To get the polynomial that is exactly divisible by g(x)=3x+2, we have to divide f(x)=3x

4

−2x

3

+3x

2

−2x+3 by g(x) and get the remainder as shown in the above image.

Hence,  

27

185

 should be subtracted from 3x

4

−2x

3

+3x

2

−2x+3 so that the resulting polynomial is exactly divisible by 3x+2.

heyyy please help me here​

Answers

Answer:

The correct answer is 111.

Step-by-step explanation:

Each time, the number increases by the number it increased by before, plus 2.

The full sequence would be:

3, 9, 17, 27 , 39, 53, 69, 89, 111.

Hope this helps:) Goodluck!

In many cases, lenders allow homeowners to include their homeowners insurance premium with their monthly mortgage payment. tim’s home is worth $279,500. if his homeowners insurance premium is $0.33 per $100, how much is added to his monthly mortgage payment for insurance? a. $7.69 b. $76.86 c. $92.35 d. $922.35 please select the best answer from the choices provided a b c d

Answers

The amount that is added to his monthly mortgage payment for insurance is b. $76.86.

What is a Mortgage?

This refers to the amount of money that is paid for the downpayment for a piece of property.

Given that the worth of the home is $279,000

premium is $0.33 per $100.

1 month premium on $1 is $0.0275/100

Now, one month's premium is calculated as:

$0.0275/100 * 279,000

=$76.86

Read more about mortgages here:

https://brainly.com/question/26654897

#SPJ4


What is the quotient of 1.728 x 109 and 4.8 x 106 expressed in scientific notation?

Answers

Answer:

4.1386 X 10^3

Step-by-step explanation:

since,

1.728 x 109 = 187 .48

4.8 X 106 = 0.04528 (approx. 0.0453)

187.48 ÷ 0.0453 = 4138.6

4138.6 = 4.1386 X 10^3

What is the quotient of [tex]1.728*10^9 and, 4.8*10^6[/tex] expressed in scientific notation?

[tex]\frac{1.728*10^9}{4.8*10^6}[/tex]  Divide

[tex]\frac{1.728}{4.8} * \frac{10^9}{10^6}[/tex] Divide numbers and 10's separately.

[tex]0.36*10^3[/tex] Subtract exponents.

[tex](3.6*10^-^1 )*10^3[/tex] Rewrite as a number bigger than 1

[tex]3.6*(10^-^1*10^3)[/tex] Associate

       The answer is

[tex]3.6*10^2[/tex] Add exponents

Two exterior angles of a triangle are 112 and 98. Find the third exterior angle.

Answers

Answer: 150°

Step-by-step explanation:

let me know if correct!

150 degrees is the answer

Mathew wants to rent a jet ski for a day. it will cost him $125 per hour plus a $10 fee for a life jacket. let h = the number of hours mathew rents the jet ski. write an expression that shows the total amount he will pay. h + part b evaluate the expression you wrote to find the amount mathew will pay if he rents the jet ski for 3 hours. $

Answers

Answer: A. 125h + 10 B. $385

Step-by-step explanation:

let h = the number of hours Mathew rents the jet ski

a. 125h + 10

B. Plug in 3 for h

125(3) + 10

375 + 10

⇒ $385

Answer:

F(h)= 125h+10

F(3)=125(3)+10

F(3)=375+10

F(3)=385

Step-by-step explanation:

So sorry if this is incorrect. I hope I could help!

Which of the following sets of numbers could represent the three sides of a triangle?
O {11, 16, 25}
O {7,13,20}
O {7, 11, 20}
O {11, 15, 27}

Answers

Answer:

{11, 16, 25}

Step-by-step explanation:

This is because whenever you add two sides of a triangle, it must be greater than the third side.

Please help me on this!

Answers

By evaluating, we will see that for x = 7 h(x) exceeds g(x).

For which value of x does h(x) first exceed g(x)?

We have:

h(x) = 2*(2)^x

g(x) = 22x - 4

Now we can evaluate both function in the different options:

h(5) = 2*(2)^5 = 64

g(5) = 22*5 - 4 = 106

We can see that for x = 5, h(x) does not exceed g(x), let's try the next one.

h(6) = 2*(2)^6 = 128

g(6) = 22*6 - 4 = 128

So, for x = 6 both functions have the same output, then for x = 7 we will see that h(x) exceeds g(x).

The correct option is B.

If you want to learn more about functions:

https://brainly.com/question/4025726

#SPJ1

anyone knows the answer?!​

Answers

Answer:

p = -6.32, 6.32

Step-by-step explanation:

[tex]3 {p}^{2} - 120 = 0[/tex]

[tex] {p}^{2} - 40 = 0[/tex]

[tex] {p}^{2} = 40[/tex]

p = -2√10, 2√10

p= -6.32, 6.32

a bucket is 12cm in diameter at the top , 8cm in diameter at the bottom and 4cm deep.calculate uts volume in cm cubein terms of pi​

Answers

Answer:

that is 12x8x4= 384

that should be the final answer

helppppp its in the picture

Answers

Answer:

129 m²

Step-by-step explanation:

S1 = ½ × 6 × 4 = 12 m²

S2 = 5 × 7 = 35 m²

S = 2 × S1 + 3 × S2

= 2 × 12 + 3 × 35

= 24 + 105

= 129 m²

HOPE THIS HELPS AND HAVE A NICE DAY <3

What is the coefficient of x in 2x+3?
0
2
3
5

Answers

Answer:

The coefficient of x becomes 4. Hope it helps you

Step-by-step explanation:

Answer: 2

Step-by-step explanation: Mark me brainliest

A set of data has a mean of 0. 5 and a standard deviation of 0. 1. A data point of the set has a z-score of 2. 5. What does a z-score of 2. 5 mean? The data point is 0. 01 standard deviations away from 2. 5. The data point is 0. 01 standard deviations away from 0. 5 The data point is 2. 5 standard deviations away from 0. 1. The data point is 2. 5 standard deviations away from 0. 5.

Answers

Answer:

The answer is D

Step-by-step explanation: I just took the test on edge

What is the value of 6 n=2 (n-1)!/2

Answers

[tex]\bold{ANSWER:}[/tex]
76.5

[tex]\bold{EXPLANATIONS:}[/tex]

Answer: d- 76.5

Step-by-step explanation:

me podrían ayudar con esto?​

Answers

Answer:

0

Step-by-step explanation:

This would be basically be zero/0 because anything times 0 is just nothing.

PLEASE ANSWER AS FAST as u can

Answers

Answer:

5.4 cm

Step-by-step explanation:

[tex] c =\pi d[/tex]

[tex] \implies 17 =\pi d[/tex]

[tex] \implies \frac{17}{\pi} =d[/tex]

[tex] \implies \frac{17}{3.14} =d[/tex]

[tex] \implies 5.41401274 =d[/tex]

[tex] \implies d\approx 5.4\: cm[/tex]

Hope it helps you in your learning process

Find the base and height of this parallelogram.

Answers

Divide the parallelogram into two triangles, assume that our e diagonal is the "base" for both new triangles. What's the height of that triangle? Use the sine function. It's (f/2) * sin (angle)! The area of the triangle is equal to our "base" e times height: e * (f/2) * sin (angle)

The base is 12 feet long, the height is 17 feet long.

How To Solve: https://brainly.com/question/544471

Credit to: naǫ

Can you answer this for me ty :)

Answers

Answer: 20ft

Explanation: To find the perimeter, double the given side lengths and add them together. This brings you to a perimeter of 80 feet for Surinder’s garden.

From there, subtract 60 from 80, which gives you 20, and you have your answer. Hope this helps!

What is the coefficient of the x2-term of the algebraic expression 2 x 4 + x squared minus x + 7?
-1
1
0
2

Answers

Answer:

1

Step-by-step explanation:

If x doesnt have a number in front of it the coefficient is 1.     x^2 + 3x + 7 = 1x^2 + 3x + 7 /// same thing

Answer:

i believe its B. 1

Step-by-step explanation:

HELP ASAP PLSSSSS
(picture of graph is below)
Below are the data collected from two random samples of 500 American adults on the number of hours they spend doing leisure and sports activities per day (rounded to the nearest hour):


Number of hours spent doing leisure and sports activities per day 1 2 3 4 5
Sample A: Number of adults 70 90 135 140 65
Sample B: Number of adults 80 80 130 135 75


Dan concludes that adults spend a mean of 3 hours each day doing leisure and sports activities. Bret thinks the mean is 4 hours. Who is correct—Dan or Bret? Explain your answer in two or three sentences. Make sure to use facts to support your answer.

Answers

Answer:

Dan is correct

Step-by-step explanation:

Mean = the sum of all data values divided by the total number of data values

Number of adults in Sample A:

= 70 + 90 + 135 + 140 + 65 = 500

Mean of Sample A:

= [ (1 × 70) + (2 × 90) + (3 × 135) + (4 × 140) + (5 × 65) ] ÷ 500

= 1540 ÷ 500

= 3.08

Number of adults in Sample B:

= 80 + 80 + 130 + 135 + 75 = 500

Mean of Sample A:

= [ (1 × 80) + (2 × 80) + (3 × 130) + (4 × 135) + (5 × 75) ] ÷ 500

= 1545 ÷ 500

= 3.09

Mean of the two samples:

= (3.08 + 3.09) ÷ 2

= 3.085 hours

= 3 hours (nearest hour)

The mean of Sample A is 3.08 hours and the mean of Sample B is 3.09 hours, so the mean of the entire sample is 3.085 hours.  If we round this to the nearest hour, then the mean is 3 hours.  Therefore, Dan is correct in concluding that the adults spend a mean of 3 hours each day doing leisure and sports activities.  

pls help me out i cannot solve this, this is khan academy btw

Answers

Answer:

Step-by-step explanation:

(0,5)  (3, 1)

(1 - 5)/(3 - 0) = -4/3

y - 5 = -4/3(x - 0)

y - 5 = -4/3x + 0

y = -4/3x + 5

y < -4/3x + 5

6.3+n=6.3

can you help me solve this awnser

Answers

Answer:

n=0

Step-by-step explanation:

subtract 6.3 from both sides

Answer:

your answer is N=0

All you have to do is subtract 6.3 from both sides and you'll be left with N=0

HURRRYYYY PLEASE ANSWER SUPER QUICK The table shows the relationship between x and y.
X
2
6
10
y
8
12
16
20
Select from the drop-down menu to correctly complete the statement.
The value of the dependent variable y is always
the value of the independent variable x.
Choose.
one-half
times
2
5
Next ›
2 more than

Answers

Answer:

2 times

Step-by-step explanation:

According to the table, each value of y (dependent variable) is twice the corresponding value of x (independent variable).

This can be shown as

y = 2x

Correct choice is 2 times



Which is the graph of the function f(x) = -√x

Answers

This is the graph, I hope it helps

If x=y, why is 2 is not equal to 1.

Answers

Step-by-step explanation:

this question is kinda tricky

A quantity of bacteria doubles every hour. If you start with 100 bacteria cells, the number of cells you will have at any time can be found by the formula c = 100(2)x. How many hours will it take to have 1,000,000 bacteria? Round to the nearest whole number.

Answers

Answer:

10,000 hours

Step-by-step explanation:

10,000 x 100 = 1,000,000

Hope this helps!

An item has a listed price of $50. If the sales tax rate is 7%, how much is the sales tax (in dollars)?
Sales tax:

Answers

Answer:  $3.50

Work Shown:

7% = 7/100 = 0.07

7% of $50 = 0.07*50 = 3.50

Extra info: The total price including tax would be 50+3.50 = 53.50 dollars. However, it seems like your teacher is only interested in the tax only.

A softball has a volume of about 29 cubic inches. Find the radius of the softball. Round your answer to the nearest tenth.

The radius of the softball is about _ inches.

Answers

The geometry of the softball is a sphere. Then the radius of the softball will be 1.91 inches.

What is Geometry?

It deals with the size of geometry, region, and density of the different forms both 2D and 3D.

A softball has a volume of about 29 cubic inches.

Then the radius of the softball will be

The geometry of the softball is a sphere.

Then the volume of the sphere is given as

[tex]\rm V = \dfrac{4}{3} \pi r^3[/tex]

where r is the radius of the softball.

Then we have

[tex]\rm r^3 = \dfrac{3V}{ 4 \pi}\\\\r^3 = \dfrac{3 \times 29}{4 \times \pi}\\\\r^3 = \dfrac{87}{12.5664}\\\\r^3 = 6.9232\\\\r \ \ = \sqrt[3]{6.9232}\\\\r \ \ = 1.9059 \approx 1.91 \ in.[/tex]

More about the geometry link is given below.

https://brainly.com/question/7558603

#SPJ1

A town has a population of 13,000 and grows at 5% every year what will the population be after 11 years to the nearest whole number

Answers

This question is based on the concept of
amount of interest. Therefore, 22,234 would be
the population after 11 years, to the nearest
whole number.
Given:
A town has a population of 13000 and grows at
5% every year.
We need to determined the population after 11
years, to the nearest whole number.
We know that,
Formula =A = P(1+ r)
Where, p be the population. r is the rate and t is
the time.
P= 13000,
r= 5%
†= 11
Now put all these values in formula.
We get,
5
A = 13000(1 +
A
= 13000(1 + 0.05)11
A
= 1300022, 234.41(0.95)
Other Questions
find the value of z why the left ventricle is thicker than the right ventricle? What is the range of possible values for x?The diagram is not to scale.a) 12 < x < 48b) 0 < x < 10c) 10 < x < 50d) 10 < x < 43 Ultraviolet light from a distant star is traveling at 3.0 108 m/s. how long will it take for the light to reach earth if it must travel 4.0 1013 km? 2.7 102 hours 2.2 103 hours 3.7 104 hours 1.3 105 hours 7Calories fuel the body, and junk food tends to be high in calories. Why isn't junk food healthy?O A. There are very few nutrients with those calories..It is healthy for those who have a high metabolism..High calorie food is often cheaper than nutritious food.OD.Calories need to be in a very specific range to be healthy.ResetNext A scuba diver is what type of resource ? A. Natural resource (N)B. Human resource (H)C. Capital good (C). Which of the following might require someone to mediate? Why? Which of the following best describes the average amount of stored data per user for the first eight years of the application existence A nurse is explaining prep to a patient. the nurse tells the patient that: prep may reduce the risk of hiv infection up to 50%. prep should be taken by anyone who injects illicit drugs. prep eliminates the need to use other hiv prevention strategies. prep is a prescription pill that contains two medications used to treat hiv. HELP The question is in the picture, I will give 5 stars! Who was Hernando Corts and what was his role in ending the Aztec civilization? I'd like 25% of my order to be 3 Monstecupcakes, and the rest Tropical. Which statement is true? The value of 8,590 4,586 is half the value of 2 x (8,590 4,586). The value of 2,674 x 5,480 + 5 is 5 times the value of 2,674 + 5,480. The value of 6,432 x (1,860 6) is 6 less than the value of 6,432 x 1,860. The value of 2,876 x (9,280 + 8) is 8 more than the value of 2,876 x 9,280. Which of the following does NOT affect when people make a purchase?2 pointsA. Time pressureB. Discount salesC. Certain holidaysD. Store atmosphereWhich of the following post-secondary education options operates as an educational nonprofit organization who receives funding through tuition, donations and endowments?2 pointsA. Community collegesB. Public universitiesC. Private universitiesD. Vocational and trade schoolsWhich of the following questions measures a persons activity?2 pointsA. Do you agree the U.S. medical system needs reform?B. How important is it to you to try new food?C. How often do you visit shopping malls?D. Do you enjoy reading? Security is a major concern with m-commerce. How can m-commerce software ensure the security of transmissions and that the transaction is being made with the intended party Please help with #6. The answer has to include double letters ( #1 is an example) If a ball is thrown straight upward with a velocity of 9. 98 m/s, what is the velocity of the ball as it passes a treetop 4. 88 m tall? The principles of operant conditioning are best illustrated by. a net force, the magnitude of which is 2800 n, accelerates a 1250-kg vehicle for 8.0 s. the vehicle travels 80.0 m during this time. what power output does this represent? Help me please!!!!!!!!